LSAT and Law School Admissions Forum

Get expert LSAT preparation and law school admissions advice from PowerScore Test Preparation.

 Administrator
PowerScore Staff
  • PowerScore Staff
  • Posts: 8929
  • Joined: Feb 02, 2011
|
#104123
Complete Question Explanation

Parallel Flaw. The correct answer choice is (A).

Answer choice (A): This is the correct answer choice.

Answer choice (B):

Answer choice (C):

Answer choice (D):

Answer choice (E):

This explanation is still in progress. Please post any questions below!
User avatar
 zoezoe6021
  • Posts: 27
  • Joined: Dec 29, 2023
|
#104613
I don't see the flaw here.

Stimulus:
Premise: /gain→/follow
Conclusion:follow→gain
User avatar
 Jeff Wren
PowerScore Staff
  • PowerScore Staff
  • Posts: 451
  • Joined: Oct 19, 2022
|
#104685
Hi Zoe,

This question (like so many) is really a test of very careful reading and being very precise with the terms in an argument and noticing when those terms change.

The stimulus starts by stating that "people would not follow a leader if they felt that there was nothing they could gain by following that leader" (my emphasis). The key word here is "felt." Just because people feel that there is or isn't something to gain doesn't actually mean that they are correct.

This statement is conditional.

If people didn't feel that there was something to gain, then they would not follow a leader.

The contrapositive would be:

If they did follow a leader, then they felt that there was something to gain.

The conclusion of the argument isn't that the people felt like there was something to gain, but that they did in fact gain/benefit ("the leaders brought some good to their followers"), which is not necessarily true, of course.

Answer A parallels this flaw by going from what people believe is true about certain theories to then concluding that there is some actual truth to these theories.
User avatar
 lemonade42
  • Posts: 95
  • Joined: Feb 23, 2024
|
#106767
Hi, I'm having trouble seeing how the conclusion is follow ----> gain.
I saw it as gain ---> follow
So I was looking for an MN mistake since the statement in the stimulus was "nothing to gain ---> not follow"
User avatar
 Jeff Wren
PowerScore Staff
  • PowerScore Staff
  • Posts: 451
  • Joined: Oct 19, 2022
|
#106834
Hi lemonade,

So the conclusion isn't actually worded as a conditional statement, so it's understandable how it can be confusing to understand/diagram.

By stating that "even leaders who are evil or incompetent bring some good," the implication is that all leaders (even the evil and incompetent ones) bring some good.

Your diagram, (if there's a gain, then they will follow) doesn't match the meaning of the conclusion because it would be possible to follow leaders even without getting a gain/some good.

The flaw in the conclusion isn't a Mistaken Negation. Instead, it's the subtle shift in wording/terms from feeling that there is something to gain in the premise to actually gaining something in the conclusion.

Get the most out of your LSAT Prep Plus subscription.

Analyze and track your performance with our Testing and Analytics Package.